www.vorhilfe.de
- Förderverein -
Der Förderverein.

Gemeinnütziger Verein zur Finanzierung des Projekts Vorhilfe.de.
Hallo Gast!einloggen | registrieren ]
Startseite · Mitglieder · Impressum
Forenbaum
^ Forenbaum
Status VH e.V.
  Status Vereinsforum

Gezeigt werden alle Foren bis zur Tiefe 2

Navigation
 Startseite...
 Suchen
 Impressum
Das Projekt
Server und Internetanbindung werden durch Spenden finanziert.
Organisiert wird das Projekt von unserem Koordinatorenteam.
Hunderte Mitglieder helfen ehrenamtlich in unseren moderierten Foren.
Anbieter der Seite ist der gemeinnützige Verein "Vorhilfe.de e.V.".
Partnerseiten
Weitere Fächer:

Open Source FunktionenplotterFunkyPlot: Kostenloser und quelloffener Funktionenplotter für Linux und andere Betriebssysteme
Forum "Integration" - Integration von 1/x
Integration von 1/x < Integration < Funktionen < eindimensional < reell < Analysis < Hochschule < Mathe < Vorhilfe
Ansicht: [ geschachtelt ] | ^ Forum "Integration"  | ^^ Alle Foren  | ^ Forenbaum  | Materialien

Integration von 1/x: Frage (beantwortet)
Status: (Frage) beantwortet Status 
Datum: 13:31 Mi 04.12.2013
Autor: Herbart

Hallo,

ich habe eine recht einfache Frage, bei der ich mir trotzdem unsicher bin. Wenn ich z.B. [mm] \integral_{-1}^{1}{\frac{1}{x} dx} [/mm] bereche, ist mir klar, dass [mm] \frac{1}{x} [/mm] bei [mm] x\to0 [/mm] gegen [mm] +-\infty [/mm] läuft, je nach dem, ob ich von unten oder von oben gegen 0 gehe. Jetzt ist doch aber aufgrund von Punktsymmetrie [mm] \integral_{0}^{1}{\frac{1}{x} dx}=-\integral_{-1}^{0}{\frac{1}{x} dx}. [/mm] Ist deshalb [mm] \integral_{-1}^{1}{\frac{1}{x} dx}=0? [/mm]

MfG Herbart

        
Bezug
Integration von 1/x: Antwort
Status: (Antwort) fertig Status 
Datum: 13:44 Mi 04.12.2013
Autor: Richie1401

Hallo,

> Hallo,
>  
> ich habe eine recht einfache Frage, bei der ich mir
> trotzdem unsicher bin. Wenn ich z.B.
> [mm]\integral_{-1}^{1}{\frac{1}{x} dx}[/mm] bereche, ist mir klar,
> dass [mm]\frac{1}{x}[/mm] bei [mm]x\to0[/mm] gegen [mm]+-\infty[/mm] läuft, je nach
> dem, ob ich von unten oder von oben gegen 0 gehe. Jetzt ist
> doch aber aufgrund von Punktsymmetrie
> [mm]\integral_{0}^{1}{\frac{1}{x} dx}=-\integral_{-1}^{0}{\frac{1}{x} dx}.[/mm]

Nein, das stimmt so nicht.
[mm] \integral_{0}^{1}{\frac{1}{x} dx}=\infty [/mm]
Das ganz Ding existiert gar nicht. Von daher kann man es auch nicht gleichsetzen.

> Ist deshalb [mm]\integral_{-1}^{1}{\frac{1}{x} dx}=0?[/mm]
>  
> MfG Herbart

Was ist (eine) Stammfunktion von $f(x)=1/x$? Eine wäre z.B. $F(x)=ln|x|$
Den Betrag also nicht vergessen. Wenn du nun das Integral mal aufspaltest in
   [mm] \int_{-1}^{0}1/xdx+\int_0^11/xdx [/mm]
so erkennst du auch, dass das alles nicht funktionieren kann.

Bezug
        
Bezug
Integration von 1/x: Cauchyscher Hauptwert
Status: (Antwort) fertig Status 
Datum: 14:45 Mi 04.12.2013
Autor: Al-Chwarizmi


> Hallo,
>  
> ich habe eine recht einfache Frage, bei der ich  
> mir trotzdem unsicher bin. Wenn ich z.B.
> [mm]\integral_{-1}^{1}{\frac{1}{x}\ dx}[/mm] bereche, ist mir klar,
> dass [mm]\frac{1}{x}[/mm] bei [mm]x\to0[/mm] gegen [mm]+-\infty[/mm] läuft, je nach
> dem, ob ich von unten oder von oben gegen 0 gehe.  
> Jetzt ist doch aber aufgrund von Punktsymmetrie
> [mm]\integral_{0}^{1}{\frac{1}{x}\ dx}\ =\ -\integral_{-1}^{0}{\frac{1}{x}\ dx}.[/mm]
> Ist deshalb [mm]\integral_{-1}^{1}{\frac{1}{x} dx}=0\ ?[/mm]
>  
> MfG Herbart


Hallo Herbart,

im Sinne eines Riemannschen Integrals existiert  
[mm] $\integral_{-1}^{1}\frac{1}{x}\ [/mm] dx$   nicht, wie Richie schon klar gemacht hat.
Allerdings ist es für gewisse besondere Zwecke trotzdem
sinnvoll, diesem Ausdruck den Wert 0 zuzuordnen, wie
du dir das vorstellst. Man muss sich dann allerdings
bewusst sein, dass es sich dabei um eine Integral-
definition handelt, die nur unter ganz bestimmten
Vorbehalten Sinn macht. Schau dazu da nach:

[]Cauchyscher Hauptwert eines divergenten Integrals

LG ,   Al-Chwarizmi


Bezug
                
Bezug
Integration von 1/x: Frage (beantwortet)
Status: (Frage) beantwortet Status 
Datum: 15:35 Mi 04.12.2013
Autor: Herbart

Vielen Dank für eure Antworten. Ich habe mir ein anderes Integral ausgedacht, das auch Punktsymmetrie aufweist. Was wäre denn mit
[mm] \integral_{-1}^{1}{\frac{x}{x^2+\epsilon} dx} [/mm] mit [mm] \epsilon>0 [/mm] beliebig, aber fest. Hier könnte ich doch argumentieren, dass
[mm] -\integral_{-1}^{0}{\frac{x}{x^2+\epsilon} dx}=\integral_{0}^{1}{\frac{x}{x^2+\epsilon} dx}, [/mm]
weil beide integrale zwar gegen [mm] \infty [/mm] konvergieren bei [mm] \epsilon [/mm] von oben [mm] \to0, [/mm] also nicht existieren würden, aber für beliebiges [mm] \epsilon>0 [/mm] im allgemeinen [mm] <\infty [/mm] sind!
=> [mm] \integral_{-1}^{1}{\frac{x}{x^2+\epsilon} dx}=0 [/mm]
Oder liege ich wieder falsch?

MfG Herbart

Bezug
                        
Bezug
Integration von 1/x: Antwort
Status: (Antwort) fertig Status 
Datum: 15:53 Mi 04.12.2013
Autor: Al-Chwarizmi


> Vielen Dank für eure Antworten. Ich habe mir ein anderes
> Integral ausgedacht, das auch Punktsymmetrie aufweist. Was
> wäre denn mit
> [mm]\integral_{-1}^{1}{\frac{x}{x^2+\epsilon} dx}[/mm] mit
> [mm]\epsilon>0[/mm] beliebig, aber fest. Hier könnte ich doch
> argumentieren, dass
> [mm]-\integral_{-1}^{0}{\frac{x}{x^2+\epsilon} dx}=\integral_{0}^{1}{\frac{x}{x^2+\epsilon} dx},[/mm]
>  
> weil beide integrale zwar gegen [mm]\infty[/mm] konvergieren bei
> [mm]\epsilon[/mm] von oben [mm]\to0,[/mm] also nicht existieren würden, aber
> für beliebiges [mm]\epsilon>0[/mm] im allgemeinen [mm]<\infty[/mm] sind!
>  => [mm]\integral_{-1}^{1}{\frac{x}{x^2+\epsilon} dx}=0[/mm]

>  Oder
> liege ich wieder falsch?
>  
> MfG Herbart


Hallo,

für dieses Integral brauchst du dann nicht einmal den
"Trick" mit dem Cauchy-Hauptwert, denn dein Integrand
ist ja auf dem ganzen Integrationsintervall stetig, und
er stellt eine ungerade Funktion dar. In diesem Fall
gilt sogar für jedes a mit [mm] a\ge0 [/mm] :  

      [mm] $\integral_{-a}^{a}f(x)\ [/mm] dx\ =\ 0$    (falls [mm] \epsilon>0) [/mm]

Einen nachfolgenden Grenzübergang mit  [mm] \epsilon\to0 [/mm]
müsste man sich dann aber trotzdem detailliert überlegen
und klar angeben, was wirklich gemeint sein soll. Jeden-
falls lässt sich auf diesem Weg nicht zeigen, dass

     [mm] $\integral_{-a}^{a}\frac{1}{x}\ [/mm] dx\ =\ 0$

(im Riemannschen Sinne)

LG ,  Al-Chw.


Bezug
                                
Bezug
Integration von 1/x: Mitteilung
Status: (Mitteilung) Reaktion unnötig Status 
Datum: 16:12 Mi 04.12.2013
Autor: Herbart


> Jeden-
>  falls lässt sich auf diesem Weg nicht zeigen, dass
>  
> [mm]\integral_{-a}^{a}\frac{1}{x}\ dx\ =\ 0[/mm]
>  
> (im Riemannschen Sinne)
>  
> LG ,  Al-Chw.
>  

Das hatte ich auch nicht unbedingt vor. Es ging mir eher darum zu überlegen, welche Grenzfälle es geben könnte, da wir die Funktion ja für hinreichend kleine [mm] \epsilon [/mm] bel. groß [mm] (<\infty) [/mm] machen können.
Vielen Dank für deine Antwort.

MfG Herbart

Bezug
Ansicht: [ geschachtelt ] | ^ Forum "Integration"  | ^^ Alle Foren  | ^ Forenbaum  | Materialien


^ Seitenanfang ^
ev.vorhilfe.de
[ Startseite | Mitglieder | Impressum ]